LSAT and Law School Admissions Forum

Get expert LSAT preparation and law school admissions advice from PowerScore Test Preparation.

 Administrator
PowerScore Staff
  • PowerScore Staff
  • Posts: 8919
  • Joined: Feb 02, 2011
|
#26240
Complete Question Explanation
(The complete setup for this game can be found here: lsat/viewtopic.php?t=10888)

The correct answer choice is (E)

Considering the wording of each answer, this question is meant to test your understanding of the numerical limitations applicable to each group. To recap, here is what we know so far:
June15_game_1_#4_diagram_1.png
Answer choice (A) is incorrect, because it is possible that no one receives a $1k bonus. In that case, K and P would receive $3k bonuses, while L and M would receive $5k bonuses.

Answer choice (B) is incorrect, because it is possible that only Z and V receive $3k bonuses. The minimum number of variables in the $3k group is two.

Answer choice (C) is incorrect, because as many as four employees can receive $3k bonuses (those could be V, Z, L, and M, or, alternatively, V, Z, K, and P).

Answer choice (D) is incorrect, because X is the only employee who must receive a $5k bonus. Thus, the minimum number of employees who must receive a $5k bonus is one, not two.

Answer choice (E) is the correct answer choice, because neither K nor P can receive a $5k bonus. Consequently, at least one (X) and at most three (X, L, M) of the employees can receive that bonus.

Get the most out of your LSAT Prep Plus subscription.

Analyze and track your performance with our Testing and Analytics Package.